Skip to main content
Log in

Spectral Properties of Relativistic Quantum Waveguides

  • Original Paper
  • Published:
Annales Henri Poincaré Aims and scope Submit manuscript

Abstract

We make a spectral analysis of the massive Dirac operator in a tubular neighbourhood of an unbounded planar curve, subject to infinite mass boundary conditions. Under general assumptions on the curvature, we locate the essential spectrum and derive an effective Hamiltonian on the base curve which approximates the original operator in the thin-strip limit. We also investigate the existence of bound states in the non-relativistic limit and give a geometric quantitative condition for the bound states to exist.

This is a preview of subscription content, log in via an institution to check access.

Access this article

Price excludes VAT (USA)
Tax calculation will be finalised during checkout.

Instant access to the full article PDF.

Similar content being viewed by others

References

  1. Arrizabalaga, N., Le Treust, L., Raymond, N.: On the MIT bag model in the non-relativistic limit. Commun. Math. Phys. 354(2), 641–669 (2017)

    Article  ADS  MathSciNet  Google Scholar 

  2. Barbaroux, J.-M., Cornean, H.D., Le Treust, L., Stockmeyer, E.: Resolvent convergence to Dirac operators on planar domains. Ann. Henri Poincaré 20, 1877–1891 (2019)

    Article  ADS  MathSciNet  Google Scholar 

  3. Benguria, R.D., Fournais, S., Stockmeyer, E., Van Den Bosch, H.: Self-adjointness of two-dimensional Dirac operators on domains. Ann. Henri Poincaré 18, 1371–1383 (2017)

    Article  ADS  MathSciNet  Google Scholar 

  4. Berry, M.V., Mondragon, R.J.: Neutrino billiards: time-reversal symmetry-breaking without magnetic fields. Proc. R. Soc. Lond. Ser. A 412(1842), 53–74 (1987)

    Article  ADS  MathSciNet  Google Scholar 

  5. Bolte, J., Harrison, J.: Spectral statistics for the Dirac operator on graphs. J. Phys. A 36(11), 2747–2769 (2003)

    Article  ADS  MathSciNet  Google Scholar 

  6. Borrelli, W., Carlone, R., Tentarelli, L.: Nonlinear Dirac equation on graphs with localized nonlinearities: bound states and nonrelativistic limit. SIAM J. Math. Anal. 51(2), 1046–1081 (2019)

    Article  MathSciNet  Google Scholar 

  7. Borrelli, W., Carlone, R., Tentarelli, L.: A note on the Dirac operator with Kirchoff-type vertex conditions on noncompact metric graphs, Springer INdAM Series—Mathematical Challenges of Zero-Range Physics, pp. 81–104 (2021)

  8. Borrelli, W., Carlone, R., Tentarelli, L.: On the nonlinear Dirac equation on noncompact metric graphs. J. Differ. Equ. 278, 326–357 (2021)

    Article  ADS  MathSciNet  Google Scholar 

  9. Bulla, W., Trenkler, T.: The free Dirac operator on compact and noncompact graphs. J. Math. Phys. 31(5), 1157–1163 (1990)

    Article  ADS  MathSciNet  Google Scholar 

  10. Cycon, H.L., Froese, R.G., Kirsch, W., Simon, B.: Schrödinger Operators, with Application to Quantum Mechanics and Global Geometry. Springer, Berlin (1987)

    MATH  Google Scholar 

  11. Davies, E.B.: Spectral Theory and Differential Operators, Cambridge Studies in Advanced Mathematics, vol. 42. Cambridge University Press, Cambridge (1995)

    Book  Google Scholar 

  12. de Oliveira, C.R.: Quantum singular operator limits of thin Dirichlet tubes via \(\Gamma \)-convergence. Rep. Math. Phys. 66, 375–406 (2010)

    MathSciNet  Google Scholar 

  13. Duclos, P., Exner, P.: Curvature-induced bound states in quantum waveguides in two and three dimensions. Rev. Math. Phys. 7, 73–102 (1995)

    Article  MathSciNet  Google Scholar 

  14. Exner, P., Kovařík, H.: Quantum Waveguides. Springer, Berlin (2015)

  15. Exner, P., Šeba, P.: Bound states in curved quantum waveguides. J. Math. Phys. 30, 2574–2580 (1989)

    Article  ADS  MathSciNet  Google Scholar 

  16. Goldstone, J., Jaffe, R.L.: Bound states in twisting tubes. Phys. Rev. B 45, 14100–14107 (1992)

    Article  ADS  Google Scholar 

  17. Haag, S., Lampart, J., Teufel, S.: Generalised quantum waveguides. Ann. H. Poincaré 16, 2535–2568 (2015)

    Article  MathSciNet  Google Scholar 

  18. Kato, T.: Perturbation Theory for Linear Operators. Springer, Berlin (1995). Reprint of the Corr. Print. of the 2nd ed. (1980)

  19. Krejčiřík, D., Kříž, J.: On the spectrum of curved quantum waveguides. Publ. RIMS, Kyoto University 41(3), 757–791 (2005)

    Article  Google Scholar 

  20. Krejčiřík, D., Raymond, N., Royer, J., Siegl, P.: Reduction of dimension as a consequence of norm-resolvent convergence and applications. Mathematika 64, 406–429 (2018)

    Article  MathSciNet  Google Scholar 

  21. Krejčiřík, D., Šediváková, H.: The effective Hamiltonian in curved quantum waveguides under mild regularity assumptions. Rev. Math. Phys. 24, 1250018 (2012)

    Article  MathSciNet  Google Scholar 

  22. Krejčiřík, D., Zahradová, K.: Quantum strips in higher dimensions. Oper. Matrices 14, 635–665 (2020)

    Article  MathSciNet  Google Scholar 

  23. Lampart, J., Teufel, S.: The adiabatic limit of Schrödinger operators on fibre bundles. Math. Anal. 367, 1647–1683 (2017)

    Article  Google Scholar 

  24. Le Treust, L., Ourmières-Bonafos, T.: Self-adjointness of Dirac operators with infinite mass boundary conditions in sectors. Ann. H. Poincaré 19, 1465–1487 (2018)

    Article  MathSciNet  Google Scholar 

  25. Lieb, E.H., Seiringer, R.: The Stability of Matter in Quantum Mechanics. Cambridge University Press, New York (2009)

    Book  Google Scholar 

  26. Lotoreichik, V., Ourmières-Bonafos, T.: A sharp upper bound on the spectral gap for graphene quantum dots. Math. Phys. Anal. Geom. (2019)

  27. Pedersen, J.G., Gunst, T., Markussen, T., Pedersen, T.G.: Graphene antidot lattice waveguides. Phys. Rev. B 86, 245410 (2012)

    Article  ADS  Google Scholar 

  28. Rabinovich, V.S.: Boundary problems for three-dimensional Dirac operators and generalized MIT bag models for unbounded domains. Russ. J. Math. Phys. 27, 500–516 (2020)

    Article  MathSciNet  Google Scholar 

  29. Reed, M., Simon, B.: Methods of Modern Mathematical Physics, IV. Analysis of Operators. Academic Press, New York (1978)

  30. Sabirov, K.K., Babajanov, D.B., Matrasulov, D.U., Kevrekidis, P.G.: Dynamics of Dirac solitons in networks. J. Phys. A Math. Theor. 51(43), 435203 (2018)

  31. Thaller, B.: The Dirac Equation. Springer, Berlin (1992)

    Book  Google Scholar 

  32. Tretter, Ch.: Spectral Theory of Block Operator Matrices and Applications. Imperial College Press, London (2008)

    Book  Google Scholar 

  33. Wachsmuth, J., Teufel, S.: Effective Hamiltonians for constrained quantum systems. Mem. Am. Math. Soc. 230(1083) (2013)

  34. Yusupov, J.R., Sabirov, K.K., Asadov, Q.U., Ehrhardt, M., Matrasulov, D.U.: Dirac particles in transparent quantum graphs: tunable transport of relativistic quasiparticles in branched structures. Phys. Rev. E 101(6), 062208, 8 (2020)

Download references

Acknowledgements

The research of D.K. was partially supported by the EXPRO grant No. 20-17749X of the Czech Science Foundation (GACR). W.B. is member of Gruppo Nazionale per l’Analisi Matematica, la Probabilità e le loro Applicazioni (GNAMPA) of the Istituto Nazionale di Alta Matematica (INdAM).

Author information

Authors and Affiliations

Authors

Corresponding author

Correspondence to William Borrelli.

Additional information

Communicated by Jan Derezinski.

Publisher's Note

Springer Nature remains neutral with regard to jurisdictional claims in published maps and institutional affiliations.

Appendix A. Proof of some technical results

Appendix A. Proof of some technical results

In this section, we collect the proofs of some technical results stated in the paper, in order to simplify the overall presentation.

Proof of Proposition 10 and Corollary 11

The multiplication operators by \(\sigma _1\) and \(\sigma _3\) are bounded and self-adjoint in \(L^2\big ((-1,1),{\mathbb {C}}^2\big )\) thus \({\mathcal {T}}(k,m)\) is self-adjoint if and only if \({\mathcal {T}}_0\) is self-adjoint. An integration by parts easily yields that \({\mathcal {T}}_0\) is symmetric and by definition, one has

$$\begin{aligned}&\mathrm {dom}\left( {\mathcal {T}}_0^*\right) = \Big \{ u \in L^2\big ((-1,1),{\mathbb {C}}^2\big ) :\exists \ w \in L^2\big ((-1,1),{\mathbb {C}}^2\big ) \text { such that }\\&\quad \forall \ v \in \mathrm {dom}\left( {\mathcal {T}}_0\right) , \ \langle u,{\mathcal {T}}_0 v\rangle _{L^2((-1,1),{\mathbb {C}}^2)} = \langle w,v\rangle _{L^2((-1,1),{\mathbb {C}}^2)}\Big \}. \end{aligned}$$

For every \(v\in {\mathcal {D}}:= C_0^\infty \big ((-1,1),{\mathbb {C}}^2\big )\) and \(u \in \mathrm {dom}\left( {\mathcal {T}}_0^*\right) \), there holds

$$\begin{aligned} \langle {\mathcal {T}}_0^*u,v\rangle _{L^2((-1,1),{\mathbb {C}}^2)} = \langle u,{\mathcal {T}}_0 v\rangle _{L^2((-1,1),{\mathbb {C}}^2)}&= \langle u, -i \sigma _2 v'\rangle _{L^2((-1,1),{\mathbb {C}}^2)}\\&= \langle u, i \overline{\sigma _2v'}\rangle _{{\mathcal {D}}',{\mathcal {D}}} \\&= \langle -i \sigma _2 u', {\overline{v}}\rangle _{{\mathcal {D}}',{\mathcal {D}}}\\&= \langle {\mathcal {T}}_0^*u,{\overline{v}}\rangle _{{\mathcal {D}}',{\mathcal {D}}}, \end{aligned}$$

where \(\langle \cdot ,\cdot \rangle _{{\mathcal {D}}',{\mathcal {D}}}\) is the duality bracket of distributions. In particular, we know that \({\mathcal {T}}_0^*u = -i \sigma _2 u' \in L^2\big ((-1,1),{\mathbb {C}}^2\big )\) thus we get \(u\in H^1\big ((-1,1),{\mathbb {C}}^2\big )\). Moreover, if \(v \in \mathrm {dom}\left( {\mathcal {T}}_0\right) \), there holds

$$\begin{aligned} \langle {\mathcal {T}}_0^*u,v\rangle _{L^2((-1,1),{\mathbb {C}}^2)}&= \langle -i \sigma _2 u',v\rangle _{L^2((-1,1),{\mathbb {C}}^2)} \\&= \langle u,-i \sigma _2 v'\rangle _{L^2((-1,1),{\mathbb {C}}^2)} + \Big [\langle -i \sigma _2 u,v\rangle _{{\mathbb {C}}^2}\Big ]_{-1}^1\\&= \langle u,{\mathcal {T}}_0 v\rangle _{L^2((-1,1),{\mathbb {C}}^2)} - u_2(1)\overline{v_1}(1) + u_1(1)\overline{v_2}(1)\\ {}&\qquad + u_2(-1)\overline{v_1}(-1) - u_1(-1)\overline{v_2}(-1). \end{aligned}$$

Since \(v\in \mathrm {dom}\left( {\mathcal {T}}_0\right) \), we obtain

$$\begin{aligned} 0 = - (u_2(1) + u_1(1))\overline{v_1}(1) + (u_2(-1) -u_1(-1))\overline{v_1}(-1). \end{aligned}$$

This holds for any \(v\in \mathrm {dom}\left( {\mathcal {T}}_0\right) \), so that \(u_2(\pm 1) = \mp u_1(\pm 1)\) and \(v \in \mathrm {dom}\left( {\mathcal {T}}_0\right) \). In particular \({\mathcal {T}}_0^* = {\mathcal {T}}_0\). Observe that, by the closed graph theorem, \(\mathrm {dom}\left( {\mathcal {T}}(k,m)\right) \) is continuously embedded in \(H^1\big ((-1,1),{\mathbb {C}}^2\big )\) which itself is compactly embedded in \(L^2\big ((-1,1),{\mathbb {C}}^2\big )\). Thus, \({\mathcal {T}}(k,m)\) has compact resolvent.

Let us prove Point (i) by picking \(u\in \mathrm {dom}\left( {\mathcal {T}}(k,m)\right) \) and considering

$$\begin{aligned} \begin{aligned} \Vert {\mathcal {T}}(k,m)u\Vert ^2&=\Vert u'\Vert _{L^2((-1,1),{\mathbb {C}}^2)}^2 + (m^2+k^2) \Vert u\Vert _{L^2((-1,1),{\mathbb {C}}^2)}^2 \\&\quad + 2 mk \mathfrak {R}\big (\langle \sigma _3 u,\sigma _1 u\rangle _{L^2((-1,1),{\mathbb {C}}^2)}\big ) \\&\quad + 2m \mathfrak {R}\big (\langle -i\sigma _2 u',\sigma _3 u\rangle _{L^2((-1,1),{\mathbb {C}}^2)}\big )\\&\quad + 2k \mathfrak {R}\big (\langle -i\sigma _2 u',\sigma _1 u\rangle _{L^2((-1,1),{\mathbb {C}}^2)}\big ). \end{aligned} \end{aligned}$$
(69)

We rewrite (69), arguing as follows. Using the anti-commutation rules of Pauli matrices and the boundary condition, we get

$$\begin{aligned} 2 \mathfrak {R}\big (\langle \sigma _3 u,\sigma _1 u\rangle _{L^2((-1,1),{\mathbb {C}}^2)}\big ) = 2 \mathfrak {R}\big (\langle -i\sigma _2 u',\sigma _1 u\rangle _{L^2((-1,1),{\mathbb {C}}^2)}\big ) = 0 \end{aligned}$$

and

$$\begin{aligned} 2 \mathfrak {R}\big (\langle -i\sigma _2 u',\sigma _3 u\rangle _{L^2((-1,1),{\mathbb {C}}^2)}\big ) = \Vert u(1)\Vert _{{\mathbb {C}}^2}^2 + \Vert u(-1)\Vert _{{\mathbb {C}}^2}^2. \end{aligned}$$
(70)

In particular, we obtain

$$\begin{aligned} \Vert {\mathcal {T}}(k,m) u\Vert _{L^2((-1,1),{\mathbb {C}}^2)}^2&= \Vert u'\Vert _{L^2((-1,1),{\mathbb {C}}^2)}^2 + (m^2+k^2) \Vert u\Vert _{L^2((-1,1),{\mathbb {C}}^2)}^2 \\ {}&\qquad + m (\Vert u(1)\Vert _{{\mathbb {C}}^2}^2 + \Vert u(-1)\Vert _{{\mathbb {C}}^2}^2)\\ {}&\ge (m^2+k^2) \Vert u\Vert _{L^2((-1,1),{\mathbb {C}}^2)}^2. \end{aligned}$$

Hence, by the min–max principle (see Proposition 6), if \(\lambda \in Sp({\mathcal {T}}(k,m))\), we get \(|\lambda |\ge \sqrt{m^2+k^2}\). Moreover, the last inequality is strict. Indeed, if u is an eigenfunction of \({\mathcal {T}}(k,m)\) associated with an eigenvalue \(\lambda \) such that \(|\lambda | = \sqrt{m^2 +k^2}\) we necessarily get that u is a constant \({\mathbb {C}}^2\)-valued function on \((-1,1)\) satisfying the boundary conditions given in (15). It is a contradiction because it implies that \(u = 0\) identically. Hence, \({{\,\mathrm{Sp}\,}}({\mathcal {T}}(k,m)) \cap [-\sqrt{m^2+k^2},\sqrt{m^2+k^2}] = \emptyset \) and Point (i) is proved.

Now, let \(\lambda \in {{\,\mathrm{Sp}\,}}({\mathcal {T}}(k,m))\) and pick an associated eigenfunction \(u = (u_1,u_2)^\top \in \mathrm {dom}\left( {\mathcal {T}}(k,m)\right) \). There holds

$$\begin{aligned} \left\{ \begin{array}{rcl} mu_1+ku_2-u'_2 &{}=&{} \lambda u_1\,,\\ ku_1+u'_1-mu_2&{}= &{} \lambda u_2\,. \end{array}\right. \end{aligned}$$
(71)

The second equation gives \((m+\lambda )u_2=ku_1+u'_1\) and multiplying the first line by \((\lambda +m)\), we get

$$\begin{aligned} -u''_1=Eu_1\,,\qquad E:=\lambda ^2-(m^2+k^2)\,. \end{aligned}$$

Recall that \(m\ge 0\) and that by Point (i), we have \(E>0\) for all \(k\in {\mathbb {R}}\). Thus, we find

$$\begin{aligned} u_1(t)=\alpha \cos \big (\sqrt{E}(t+1)\big ) +\beta \sin \big (\sqrt{E}(t+1)\big ), \end{aligned}$$

for some constants \(\alpha ,\beta \in {\mathbb {C}}\) and as \(m+\lambda \ne 0\) we get

$$\begin{aligned}&u_2(t)=\frac{1}{\lambda +m}\cos \big (\sqrt{E}(t+1)\big )\big (k\alpha +\sqrt{E}\beta \big )\\&\quad +\frac{1}{\lambda +m}\sin \big (\sqrt{E}(t+1)\big )\big (k\beta -\sqrt{E}\alpha \big ). \end{aligned}$$

The boundary condition at \(t=-1\) gives

$$\begin{aligned} (m+\lambda -k)\alpha -\sqrt{E}\beta = 0\,. \end{aligned}$$

The boundary condition at \(t=1\) gives

$$\begin{aligned}&\big ((m+\lambda +k)\cos (2\sqrt{E}) - \sqrt{E}\sin (2\sqrt{E})\big ) \alpha + \big ((m+\lambda +k)\sin (2\sqrt{E}) +\\&\quad \sqrt{E}\cos (2\sqrt{E})\big )\beta = 0 . \end{aligned}$$

To obtain a nonzero eigenfunction u, there has to hold

$$\begin{aligned} 0 = \begin{vmatrix} m+\lambda -k&-\sqrt{E}\\ (m+\lambda +k)\cos (2\sqrt{E}) - \sqrt{E}\sin (2\sqrt{E})&(m+\lambda +k)\sin (2\sqrt{E}) + \sqrt{E}\cos (2\sqrt{E}) \end{vmatrix}. \end{aligned}$$

Computing the determinant, we are left with the implicit equation

$$\begin{aligned} m \sin (2\sqrt{E}) + \sqrt{E}\cos (2\sqrt{E}) = 0. \end{aligned}$$
(72)

In particular, it yields that the spectrum of \({\mathcal {T}}(k,m)\) is symmetric with respect to the origin and we remark that when \(m = k = 0\), we necessarily have \(\sqrt{E} = |\lambda |= p \frac{\pi }{4}\) (with \(p\in {\mathbb {N}}\)), and that in this case, a normalized eigenfunction associated with \(\lambda = \pm \frac{\pi }{4}\) is given by

$$\begin{aligned} u_k^\pm (t) = \frac{1}{2}\cos \left( k\frac{\pi }{4}(t+1)\right) \begin{pmatrix}1\\ 1\end{pmatrix} \pm \frac{1}{2} \sin \left( k\frac{\pi }{4}(t+1)\right) \begin{pmatrix}1\\ -1\end{pmatrix}, \end{aligned}$$

which proves Corollary 11.

Remark that for \(m > 0\), a solution E to (72) verifies \(\cos (2\sqrt{E})\ne 0\), and we obtain

$$\begin{aligned} \tan (2\sqrt{E}) + \frac{\sqrt{E}}{m} = 0. \end{aligned}$$
(73)

Now, for \(p \in {\mathbb {N}}_0 = {\mathbb {N}}\cup \{0\}\), define the line segments \(I_0 := [0,\frac{\pi }{2})\) and \(I_{p+1} = ((2p+1)\frac{\pi }{2}, (2p+3)\frac{\pi }{2})\)

$$\begin{aligned} g_p : I_p \rightarrow {\mathbb {R}},\quad g_p(x) = \tan (2x) +\frac{x}{m}. \end{aligned}$$
(74)

Remark that \(g_p'(x) >0\), and in particular, the only solution to \(g_0(x) = 0\) is \(x=0\). For all \(p \ge 1\), we have

$$\begin{aligned} \lim _{x\rightarrow (2p-1)\frac{\pi }{2}^+} g_p(x) = -\infty ,\quad g_p(p\pi ) = p\frac{\pi }{m}>0. \end{aligned}$$

In particular, for all \(p\ge 1\), there is a unique solution \(x_p \in I_p\) to \(g_p(x) =0\). Moreover, it satisfies \(x_p \in \big ((2p-1)\frac{\pi }{2},p\pi \big )\). Hence, for \(p\ge 1\), \(E_p(m)\) is defined as the unique solution E to \(g_p(2\sqrt{E}) = 0\). In particular, \(E_p(m) \in ((2p-1)^2 \frac{\pi ^2}{16},p^2\frac{\pi ^2}{4})\) which proves Points (ii) and (iii).

Now, we prove (iv). Guided by (72), we define the \(C^\infty \) function

$$\begin{aligned} F :\left\{ \begin{array}{lcl} {\mathbb {R}}\times {\mathbb {R}} &{}\rightarrow &{} {\mathbb {R}}\\ (\mu ,m) &{} \mapsto &{} 2m\sin (\mu ) + \mu \cos (\mu ) \end{array}\right. . \end{aligned}$$

One remarks that \(F(\frac{\pi }{2},0) = 0\) and \(\partial _\mu F(\frac{\pi }{2},0) = \frac{\pi }{2}\). Hence, by the implicit function theorem, there exists \(\delta _1,\delta _2 > 0\) and a \(C^\infty \) function \(\mu : (-\delta _1,\delta _1) \rightarrow (\frac{\pi }{2}-\delta _2,\frac{\pi }{2}+\delta _2)\) verifying \(\mu (0) = \frac{\pi }{2}\) and such that for all \(|m|<\delta _1\), there holds \(F(\mu (m),m) = 0\). Moreover, when \(m\rightarrow 0\), there holds

$$\begin{aligned} \mu (m) = \mu (0) + \mu '(0)m + {\mathcal {O}}(m^2) = \frac{\pi }{2} + \frac{4}{\pi }m + {\mathcal {O}}(m^2). \end{aligned}$$

Necessarily, for \(m > 0\) sufficiently small, there holds \(E_1(m) = \frac{1}{4}\mu (m)^2\). Hence, when \(m\rightarrow 0\), there holds

$$\begin{aligned} E_1(m) = \frac{\pi ^2}{16} + m + {\mathcal {O}}(m^2), \end{aligned}$$

which is precisely Point (iv).

Finally, we prove (v). Once again, guided by (72), we define the \(C^\infty \) function

$$\begin{aligned} G :\left\{ \begin{array}{lcl} {\mathbb {R}}\times {\mathbb {R}} &{}\rightarrow &{} {\mathbb {R}}\\ (\mu ,\nu ) &{} \mapsto &{} 2\sin (\mu ) + \mu \nu \cos (\mu ) \end{array}\right. . \end{aligned}$$

One remarks that \(G(\pi ,0) = 0\) and \(\partial _\mu G(\pi ,0) = -2\). Hence, by the implicit function theorem, there exists \(\delta _1,\delta _2 > 0\) and a \(C^\infty \) function \(\mu : (-\delta _1,\delta _1) \rightarrow (\pi -\delta _2,\pi +\delta _2)\) verifying \(\mu (0) = \pi \) and such that for all \(|\nu |<\delta _1\) there holds \(G(\mu (\nu ),\nu ) = 0\). Moreover, when \(\nu \rightarrow 0\), there holds

$$\begin{aligned} \mu (\nu ) = \mu (0) + \mu '(0)\nu + {\mathcal {O}}(\nu ^2) = \pi -\frac{\pi }{2} \nu + {\mathcal {O}}(\nu ^2). \end{aligned}$$

Necessarily, for \(m > 0\) sufficiently large, there holds \(E_1(m) = \frac{1}{4}\mu (m^{-1})^2\). Hence, when \(m\rightarrow +\infty \), there holds

$$\begin{aligned} E_1(m) = \frac{\pi ^2}{4} -\frac{\pi ^2}{4m} + O(m^{-2}), \end{aligned}$$

which gives (v). \(\square \)

Proof of Lemma 28

Let \(u \in \mathrm {dom}\left( {\mathcal {E}}_0(\varepsilon ,m)\right) \) and remark that there holds

$$\begin{aligned} \Vert {\mathcal {C}}(\varepsilon ,m) u\Vert _{L^2(\mathsf {Str},{\mathbb {C}}^2)}^2&= \Vert (-i\sigma _1)\partial _s u + m \sigma _3 u\Vert _{L^2(\mathsf {Str},{\mathbb {C}}^2)}^2\nonumber \\&\quad + \frac{1}{\varepsilon ^2}\underset{:= A}{\underbrace{\Vert (-i\sigma _2)\partial _t u - \frac{\pi }{4}(P^+ - P^-)u\Vert _{L^2(\mathsf {Str},{\mathbb {C}}^2)}^2}}\nonumber \\&\quad + \frac{1}{\varepsilon }\underset{:=B}{\underbrace{2\mathfrak {R}(\langle (-i\sigma _1)\partial _s u, (-i\sigma _2)\partial _t u - \frac{\pi }{4}(P^+ - P^-)u\rangle _{L^2(\mathsf {Str},{\mathbb {C}}^2)})}} \nonumber \\&\quad + \frac{m}{\varepsilon }\underset{:= C}{\underbrace{2\mathfrak {R}(\langle \sigma _3 u, (-i\sigma _2)\partial _t u\rangle _{L^2(\mathsf {Str},{\mathbb {C}}^2)})}}\nonumber \\&\quad - \frac{m\pi }{4\varepsilon }\underset{:=D}{\underbrace{2\mathfrak {R}(\langle \sigma _3 u, (P^+ - P^-)u\rangle _{L^2(\mathsf {Str},{\mathbb {C}}^2)})}}. \end{aligned}$$
(75)

Now, we deal with each term appearing on the right-hand side of (75). For further use, for all \(k\ge 1\), we set \(f_k^\pm := \langle u,u_k^\pm \rangle _{L^2((-1,1),{\mathbb {C}}^2)}\) and recall that \(\Pi _k\) denotes the projector defined in (25). In particular, for all \(k\ge 1\), there holds

$$\begin{aligned} \Vert \Pi _k u\Vert _{L^2(\mathsf {Str},{\mathbb {C}}^2)}^2 = \int _{{\mathbb {R}}}\Big (|f_k^+(s)|^2 + |f_k^-(s)|^2\Big )\mathrm{d}s. \end{aligned}$$

Step 1. In this step, we analyze the term A appearing in (75). We remark that

$$\begin{aligned} (-i\sigma _2\partial _t -\frac{\pi }{4}(P^+-P^-))u = \sum _{k\ge 2} \frac{(k-1)\pi }{4}(f_k^+ u_k^+ - f_k^-u_k^-). \end{aligned}$$
(76)

In particular, it gives

$$\begin{aligned} A = \frac{\pi ^2}{16}\sum _{k\ge 2}(k-1)^2 \Vert \Pi _k u\Vert _{L^2(\mathsf {Str},{\mathbb {C}}^2)}^2. \end{aligned}$$
(77)

Step 2. A straightforward computation gives

$$\begin{aligned} -i\sigma _1\partial _s u = \sum _{k\ge 1} -i (f^-_k)' u_k^+ - i (f^+_k)'u_k^-. \end{aligned}$$

In particular, using (76), there holds

$$\begin{aligned}&\langle -i\sigma _1\partial _s u, \big (-i\sigma _2\partial _t - \frac{\pi }{4}(P^+-P^-)\big ) u\rangle _{L^2(\mathsf {Str},{\mathbb {C}}^2)} \nonumber \\&\quad = \frac{\pi }{4}\sum _{k\ge 2}(k-1)\big (-i\int _{\mathbb {R}}(f^-_k)'(s)\overline{f_k^+(s)} \mathrm{d}s + i\int _{\mathbb {R}}(f^+_k)'(s)\overline{f_k^-(s)} \mathrm{d}s\big ). \qquad \end{aligned}$$
(78)

Integrating by parts, we find

$$\begin{aligned}&\overline{-i\int _{\mathbb {R}}(f^-_k)'(s)\overline{f_k^+(s)} ds + i\int _{\mathbb {R}}(f^+_k)' \overline{f_k^-(s)} \mathrm{d}s} \\&\quad = i \int _{\mathbb {R}}\overline{(f^-_k)'(s)}f_k^+(s) \mathrm{d}s - i \int _{\mathbb {R}}\overline{(f^+_k)'(s)} f_k^-(s) \mathrm{d}s \\&\quad =-\left( -i\int _{\mathbb {R}}(f^-_k)'(s)\overline{f_k^+(s)} ds + i\int _{\mathbb {R}}(f^+_k)'(s)\overline{f_k^-(s)} \mathrm{d}s\right) \,, \end{aligned}$$

and then using (78), we get

$$\begin{aligned}&\langle -i\sigma _1\partial _s u, \big (-i\sigma _2\partial _t - \frac{\pi }{4}(P^+-P^-)\big ) u\rangle _{L^2(\mathsf {Str},{\mathbb {C}}^2)} \\&\quad = -\langle \big (-i\sigma _2\partial _t - \frac{\pi }{4}(P^+-P^-)\big ) u,-i\sigma _1\partial _s u\rangle _{L^2(\mathsf {Str},{\mathbb {C}}^2)}. \end{aligned}$$

In particular, we obtain

$$\begin{aligned} B = 2\mathfrak {R}(\langle -i\sigma _1\partial _s u, \big (-i\sigma _2\partial _t - \frac{\pi }{4}(P^+-P^-)\big ) u\rangle _{L^2(\mathsf {Str},{\mathbb {C}}^2)}) = 0. \end{aligned}$$
(79)

Step 3. In this step, we deal with the term C. Integrating by parts as in (70), we obtain:

$$\begin{aligned} C = \int _{\mathbb {R}}\vert u(s,1)\vert ^2 + \vert u(s,-1)\vert ^2 \mathrm{d}s. \end{aligned}$$
(80)

Step 4 It remains to deal with the term D. To do so, we remark that:

$$\begin{aligned} \langle \sigma _3 u, (P^+-P^-)u\rangle _{L^2(\mathsf {Str},{\mathbb {C}}^2)}&= \underset{:= \alpha }{\underbrace{\langle \Pi _1\sigma _3\Pi _1 u, (P^+-P^-)u\rangle _{L^2(\mathsf {Str},{\mathbb {C}}^2)}}}\nonumber \\&\quad + \underset{:= \beta }{\underbrace{\langle \Pi _1^\perp \sigma _3\Pi _1^\perp u, (P^+-P^-)u\rangle _{L^2(\mathsf {Str},{\mathbb {C}}^2)}}} \nonumber \\&\quad +\underset{:=\gamma }{\underbrace{\langle \Pi _1\sigma _3\Pi _1^\perp u, (P^+-P^-)u\rangle _{L^2(\mathsf {Str},{\mathbb {C}}^2)}}}\nonumber \\&\quad + \underset{:= \delta }{\underbrace{\langle \Pi _1^\perp \sigma _3\Pi _1 u, (P^+-P^-)u\rangle _{L^2(\mathsf {Str},{\mathbb {C}}^2)}}}. \end{aligned}$$
(81)

Now, in each of the next substep, we deal with the terms appearing on the right-hand side of (81).

Substep 4.1 Remark that there holds

$$\begin{aligned} \alpha&= \langle f_1^+\sigma _3 u_1^+ + f_1^-\sigma _3 u_1^-, f_1^+u_1^+ - f_1^- u_1^-\rangle _{L^2(\mathsf {Str},{\mathbb {C}}^2)}\\ {}&= \langle \sigma _3 u_1^+,u_1^+\rangle _{L^2((-1,1),{\mathbb {C}}^2)}\Vert f_1^+\Vert _{L^2({\mathbb {R}})}^2 - \langle \sigma _3 u_1^-,u_1^-\rangle _{L^2((-1,1),{\mathbb {C}}^2)}\Vert f_1^-\Vert _{L^2({\mathbb {R}})}\\ {}&\qquad - \langle \sigma _3 u_1^+,u_1^-\rangle _{L^2((-1,1),{\mathbb {C}}^2)}\langle f_1^+,f_1^-\rangle _{L^2({\mathbb {R}})} \\&\quad + \langle \sigma _3 u_1^-,u_1^+\rangle _{L^2((-1,1),{\mathbb {C}}^2)}\langle f_1^-,f_1^+\rangle _{L^2({\mathbb {R}})}. \end{aligned}$$

Thanks to (31), we get

$$\begin{aligned} \alpha = \frac{2}{\pi }\Vert \Pi _1 u\Vert _{L^2(\mathsf {Str},{\mathbb {C}}^2)}^2. \end{aligned}$$
(82)

Substep 4.2 We handle the term \(\beta \) by obtaining the following upper-bound thanks to the Cauchy–Schwarz inequality:

$$\begin{aligned} |\beta | = |\langle \Pi _1^\perp \sigma _3\Pi _1^\perp u, (P^+-P^-)u\rangle _{L^2(\mathsf {Str},{\mathbb {C}}^2)}| \le \Vert \Pi _1^\perp u\Vert _{L^2(\mathsf {Str},{\mathbb {C}}^2)}^2. \end{aligned}$$
(83)

Substep 4.3 Now, let us focus on the two off-diagonal terms \(\gamma \) and \(\delta \). A direct computation shows that

$$\begin{aligned} \langle \sigma _3 u^-_k,u^+_1 \rangle _{{\mathbb {C}}^2}=-\langle \sigma _3 u^+_k,u^-_1 \rangle _{{\mathbb {C}}^2}\,,\quad \langle \sigma _3 u^-_k,u^-_1 \rangle _{{\mathbb {C}}^2}=-\langle \sigma _3 u^+_k,u^+_1 \rangle _{{\mathbb {C}}^2}\,. \end{aligned}$$

Then, we get

$$\begin{aligned} \Pi _1\sigma _3\Pi _1^\perp u = \left( \sum _{k\ge 2}a_k f_k^+ - b_k f_k^-\right) u_1^+ + \left( \sum _{k\ge 2}b_k f_k^+ - a_k f_k^-\right) u_1^-, \end{aligned}$$

where we have set for \(k\ge 2\)

$$\begin{aligned} a_k&:= \langle \sigma _3 u_k^+,u_1^+\rangle _{L^2((-1,1),{\mathbb {C}}^2)} = \frac{4}{\pi }\frac{\sin ^2(\frac{\pi }{4}(k+1))}{(k+1)},\nonumber \\ b_k&:= \langle \sigma _3 u_k^+,u_1^-\rangle _{L^2((-1,1),{\mathbb {C}}^2)} = \frac{4}{\pi }\frac{\sin ^2(\frac{\pi }{4}(k-1))}{(k-1)}. \end{aligned}$$
(84)

Thus, we find

$$\begin{aligned} \gamma = \sum _{k\ge 2} \int _{\mathbb {R}}\langle (a_k - \sigma _1 b_k)\begin{pmatrix}f_k^+\\ f_k^-\end{pmatrix},\begin{pmatrix}f_1^+\\ f_1^-\end{pmatrix}\rangle _{{\mathbb {C}}^2}\mathrm{d}s. \end{aligned}$$
(85)

A similar computation gives

$$\begin{aligned} \delta = \sum _{k\ge 2} \int _{\mathbb {R}}\langle \begin{pmatrix}f_1^+\\ f_1^-\end{pmatrix},(a_k + \sigma _1 b_k)\begin{pmatrix}f_k^+\\ f_k^-\end{pmatrix}\rangle _{{\mathbb {C}}^2}\mathrm{d}s. \end{aligned}$$
(86)

In particular, using (85) and (86), we get

$$\begin{aligned} \gamma + \delta&=2\mathfrak {R}\Big (\sum _{k\ge 2}a_k\int _{\mathbb {R}}\langle \begin{pmatrix}f_1^+\\ f_1^-\end{pmatrix},\begin{pmatrix}f_k^+\\ f_k^-\end{pmatrix}\rangle _{{\mathbb {C}}^2}\mathrm{d}s\Big )\nonumber \\&\quad + 2i\mathfrak {I}\Big (\sum _{k\ge 2}b_k \int _{\mathbb {R}}\langle \begin{pmatrix}f_1^+\\ f_1^-\end{pmatrix},\sigma _1\begin{pmatrix}f_k^+\\ f_k^-\end{pmatrix}\rangle _{{\mathbb {C}}^2}\mathrm{d}s\Big ). \end{aligned}$$
(87)

Using (81), (82) and (87), we obtain

$$\begin{aligned} D = \frac{4}{\pi }\Vert \Pi _1 u\Vert _{L^2(\mathsf {Str},{\mathbb {C}}^2)}^2 + 2 \mathfrak {R}(\beta ) + 4 \mathfrak {R}\Big (\sum _{k\ge 2}a_k\int _{\mathbb {R}}\langle \begin{pmatrix}f_1^+\\ f_1^-\end{pmatrix},\begin{pmatrix}f_k^+\\ f_k^-\end{pmatrix}\rangle _{{\mathbb {C}}^2}\mathrm{d}s\Big ). \end{aligned}$$

In particular, using the Cauchy–Schwartz inequality, we get

$$\begin{aligned} D \le \frac{4}{\pi }\Vert \Pi _1 u\Vert _{L^2(\mathsf {Str},{\mathbb {C}}^2)}^2 + 2 |\beta | + 4\sum _{k\ge 2}\Big (|a_k|\Vert \Pi _1 u\Vert _{L^2(\mathsf {Str},{\mathbb {C}}^2)}\Vert \Pi _ku\Vert _{L^2(\mathsf {Str},{\mathbb {C}}^2)}\Big ).\nonumber \\ \end{aligned}$$
(88)

Now, let us fix \(c>0\) to be chosen later. For all \(a,b\in {\mathbb {R}}\) and \(\varepsilon >0\), we recall the elementary inequality \(ab \le \frac{c\varepsilon }{2} a^2 + \frac{1}{2c\varepsilon }b^2\) that we use to get for all \(k\ge 2\):

$$\begin{aligned} |a_k|\Vert \Pi _1 u\Vert _{L^2(\mathsf {Str},{\mathbb {C}}^2)}\Vert \Pi _ku\Vert _{L^2(\mathsf {Str},{\mathbb {C}}^2)} \le \frac{c\varepsilon }{2}a_k^2 \Vert \Pi _1 u\Vert _{L^2(\mathsf {Str},{\mathbb {C}}^2)}^2 + \frac{1}{2c\varepsilon }\Vert \Pi _k u\Vert _{L^2(\mathsf {Str},{\mathbb {C}}^2)}^2. \end{aligned}$$

Then, summing up for \(k\ge 2\), we get

$$\begin{aligned} \sum _{k\ge 2}\Big (|a_k|\Vert \Pi _1 u\Vert _{L^2(\mathsf {Str},{\mathbb {C}}^2)}\Vert \Pi _ku\Vert _{L^2(\mathsf {Str},{\mathbb {C}}^2)}\Big )&\le \frac{1}{2}c\varepsilon S\Vert \Pi _1u\Vert _{L^2(\mathsf {Str},{\mathbb {C}}^2)}^2 \nonumber \\&\quad + \frac{1}{2c\varepsilon }\Vert \Pi _1^\perp u\Vert _{L^2(\mathsf {Str},{\mathbb {C}}^2)}^2, \end{aligned}$$
(89)

where we have set \(S = \sum _{k\ge 2} a_k^2 < +\infty \) because \(a_k^2 = {\mathcal {O}}(k^{-2})\) when \(k\rightarrow +\infty \) by (84). Taking into account (83) and (89), (88) gives

$$\begin{aligned} D \le (\frac{4}{\pi }+2cS\varepsilon )\Vert \Pi _1u\Vert _{L^2(\mathsf {Str},{\mathbb {C}}^2)}^2 + 2(1+\frac{1}{c\varepsilon }) \Vert \Pi _1^\perp u\Vert _{L^2(\mathsf {Str},{\mathbb {C}}^2)}^2. \end{aligned}$$
(90)

Step 5. In this step, we conclude the proof. Using (77), (79) and (80), (75) becomes

$$\begin{aligned}&\Vert {\mathcal {C}}(\varepsilon ,m) u\Vert ^2_{L^2(\mathsf {Str},{\mathbb {C}}^2)} \\&\quad = \Vert (- i\sigma _1 \partial _s + m\sigma _3)u\Vert _{L^2(\mathsf {Str},{\mathbb {C}}^2)}^2 + \frac{\pi ^2}{16\varepsilon ^2}\sum _{k\ge 2}(k-1)^2\Vert \Pi _k u\Vert _{L^2(\mathsf {Str},{\mathbb {C}}^2)}^2\\&\qquad + \frac{m}{\varepsilon }\int _{{\mathbb {R}}} \Big (\vert u(s,1)\vert ^2 + \vert u(s,-1)\vert ^2\Big )\mathrm{d}s - \frac{m\pi }{4\varepsilon } D\\&\quad = \Vert (- i\sigma _1 \partial _s + m\sigma _3)u\Vert _{L^2(\mathsf {Str},{\mathbb {C}}^2)}^2 + \frac{\pi ^2}{16\varepsilon ^2}\sum _{k\ge 2}(k-1)^2\Vert \Pi _k u\Vert _{L^2(\mathsf {Str},{\mathbb {C}}^2)}^2\\&\qquad +\frac{1}{\varepsilon ^2}\int _{\mathbb {R}}(\mathfrak {\tau }_{m\varepsilon }(u)(s)-\mathfrak {\tau }_0(u)(s))\mathrm{d}s -\frac{m\pi }{4\varepsilon } D\\&\quad = \Vert (- i\sigma _1 \partial _s + m\sigma _3)u\Vert _{L^2(\mathsf {Str},{\mathbb {C}}^2)}^2 + \frac{\pi ^2}{16\varepsilon ^2}\sum _{k\ge 2}(k-1)^2\Vert \Pi _k u\Vert _{L^2(\mathsf {Str},{\mathbb {C}}^2)}^2\\&\qquad - \frac{\pi ^2}{16\varepsilon ^2}\Vert \Pi _1 u\Vert ^2_{L^2(\mathsf {Str},{\mathbb {C}}^2)}+\frac{1}{\varepsilon ^2}\int _{{\mathbb {R}}}(\mathfrak {\tau }_{m\varepsilon }(u)(s)-\mathfrak {\tau }_0(\Pi _1^\perp u)(s))\mathrm{d}s -\frac{m\pi }{4\varepsilon } D, \end{aligned}$$

where the quadratic forms \(\mathfrak {\tau }_{\varepsilon m}\) and \(\mathfrak {\tau }_0\) are defined in (43). Notice that in the above formula, we used the fact that

$$\begin{aligned}&\mathfrak {\tau }_0(u)(s)=\mathfrak {\tau }_0(\Pi _1 u)(s)+\mathfrak {\tau }_0(\Pi ^\perp _1 u)(s)\\&\quad =\frac{\pi ^2}{16}\Vert (\Pi _1 u)(s)\Vert ^2_{L^2(-1,1,{\mathbb {C}}^2)}+\mathfrak {\tau }_0(\Pi ^\perp _1 u)(s)\,,\qquad s\in {\mathbb {R}}\, \end{aligned}$$

and

$$\begin{aligned} \Vert \Pi _1 u\Vert ^2_{L^2(\mathsf {Str},{\mathbb {C}}^2)}=\int _{{\mathbb {R}}}\Vert (\Pi _1 u)(s)\Vert ^2_{L^2((-1,1),{\mathbb {C}}^2)}\mathrm{d}s\,. \end{aligned}$$

Using Lemma (26), this last inequality becomes

$$\begin{aligned} \Vert {\mathcal {C}}(\varepsilon ,m) u\Vert ^2_{L^2(\mathsf {Str},{\mathbb {C}}^2)} \ge \&\Vert (- i\sigma _1 \partial _s + m\sigma _3)u\Vert _{L^2(\mathsf {Str},{\mathbb {C}}^2)}^2 + \frac{\pi ^2}{16\varepsilon ^2}\Vert \Pi _1^\perp u\Vert _{L^2(\mathsf {Str},{\mathbb {C}}^2)}^2\\ {}&+ \frac{1}{\varepsilon ^2}\Big (E_1(m\varepsilon ) - \frac{\pi ^2}{16}\Big )\Vert \Pi _1 u\Vert _{L^2(\mathsf {Str},{\mathbb {C}}^2)}^2 -\frac{m\pi }{4\varepsilon } D \end{aligned}$$

and (90) yields

$$\begin{aligned} \Vert {\mathcal {C}}(\varepsilon ,m) u\Vert ^2_{L^2(\mathsf {Str},{\mathbb {C}}^2)}&\ge \Vert (- i\sigma _1 \partial _s + m\sigma _3)u\Vert _{L^2(\mathsf {Str},{\mathbb {C}}^2)}^2 \nonumber \\&\quad + \frac{1}{\varepsilon ^2}\Big (\frac{\pi ^2}{16} - \frac{m\pi }{2c} - \frac{m\pi }{2}\varepsilon \Big )\Vert \Pi _1^\perp u\Vert _{L^2(\mathsf {Str},{\mathbb {C}}^2)}^2\nonumber \\&\quad + \frac{1}{\varepsilon ^2}\Big (E_1(m\varepsilon ) - \frac{\pi ^2}{16} - m\varepsilon - \frac{m\pi cS}{2}\varepsilon ^2\Big )\Vert \Pi _1 u\Vert _{L^2(\mathsf {Str},{\mathbb {C}}^2)}^2. \end{aligned}$$
(91)

Now, we choose \(c > \frac{8m}{\pi }\) and remark that there exists \(\varepsilon _1> 0\) such that for all \(\varepsilon \in (0,\varepsilon _1)\), there holds

$$\begin{aligned} \frac{\pi ^2}{16} - \frac{m\pi }{2c} - \frac{m\pi }{2}\varepsilon > 0. \end{aligned}$$
(92)

Moreover, thanks to (iv) of Proposition 10, there exists \(\varepsilon _2\) and \(K>0\) such that for all \(\varepsilon \in (0,\varepsilon _2)\)

$$\begin{aligned} E_1(m\varepsilon ) - \frac{\pi ^2}{16} - m\varepsilon - \frac{m\pi cS}{2}\varepsilon ^2 > -K \varepsilon ^2. \end{aligned}$$
(93)

Setting \(\varepsilon _0 := \min (\varepsilon _1,\varepsilon _2)\) and taking into account (92) and (93) in (91) we obtain that for all \(\varepsilon \in (0,\varepsilon _0)\), there holds

$$\begin{aligned} K\Vert \Pi _1 u\Vert _{L^2(\mathsf {Str},{\mathbb {C}}^2)}^2 + \Vert {\mathcal {C}}(\varepsilon ,m)u\Vert _{L^2(\mathsf {Str},{\mathbb {C}}^2)}^2 \ge \Vert (-i\sigma _1\partial _s + m\sigma _3)u\Vert _{L^2(\mathsf {Str},{\mathbb {C}}^2)}^2. \end{aligned}$$

The proof of Lemma 28 is completed remarking that \(\Vert \Pi _1 u\Vert _{L^2(\mathsf {Str},{\mathbb {C}}^2)}^2 \le \Vert u\Vert _{L^2(\mathsf {Str},{\mathbb {C}}^2)}^2\). \(\square \)

Rights and permissions

Reprints and permissions

About this article

Check for updates. Verify currency and authenticity via CrossMark

Cite this article

Borrelli, W., Briet, P., Krejčiřík, D. et al. Spectral Properties of Relativistic Quantum Waveguides. Ann. Henri Poincaré 23, 4069–4114 (2022). https://doi.org/10.1007/s00023-022-01179-9

Download citation

  • Received:

  • Accepted:

  • Published:

  • Issue Date:

  • DOI: https://doi.org/10.1007/s00023-022-01179-9

Mathematics Subject Classification

Navigation